LSAT and Law School Admissions Forum

Get expert LSAT preparation and law school admissions advice from PowerScore Test Preparation.

User avatar
 Dave Killoran
PowerScore Staff
  • PowerScore Staff
  • Posts: 5853
  • Joined: Mar 25, 2011
|
#44116
Complete Question Explanation
(The complete setup for this game can be found here: lsat/viewtopic.php?t=6788)

The correct answer choice is (B)

Like the other Global questions in this game (#6 and #7), this question can be time-consuming. Remember, on many Global questions using hypotheticals is a fast and effective method of attack.

Answer choice (A) is proven incorrect by the hypothetical provided in question #6.

Answer choice (B) is the correct answer choice. If the condition in the answer choice is true, then only three sessions would be attended by the employees (two sessions per employee and two employees at each session equals three sessions of two employees each). However, there is no acceptable scenario where this can occur because none of the employees can repeat a session topic.

Answer choice (C) is proven incorrect by the following hypothetical:
D02_Game_#2_#9_diagram 1.png
Answer choices (D) and (E) are functionally identical, and therefore both are incorrect. M and S are basically interchangeable, and these two answers simply pair S with T, and then M with T. If S can pair with T, then logically M can pair with T. According to the Uniqueness Theory of Answer Choices, each correct answer choice is identifiably unique, and so any pair of functionally identical answer choices must be incorrect.
You do not have the required permissions to view the files attached to this post.
 Pragmatism
  • Posts: 68
  • Joined: Jan 11, 2018
|
#44838
So, I chose "E' for this answer choice. I reasoned, that of the three employees, Tate is the only one that is not restricted to any sessions, while the other two are to hiring and regulations. So, if T was to attend investing, then this would most certainly be false? So, while answer choice B is the right answer, does answer choice B take precedence over E for the simple reason that, while you can make E can be true, B under no circumstance would be true?

If that is the case, then why isn't the "every" in answer choice E scrutinized to the degree that would reflect this answer choice to be right as well?
 Malila Robinson
PowerScore Staff
  • PowerScore Staff
  • Posts: 296
  • Joined: Feb 01, 2018
|
#44913
Hi Pragmatism,
The question stem is saying that 4 of the answer choices Could Be True and the one correct answer choice Cannot Be True. As the Admin explained (above) there is no scenario where B could work which makes it the correct answer. As you pointed out, E could be true so it is not the correct answer.
Hope that helps!
-Malila
 hope
  • Posts: 84
  • Joined: Feb 13, 2018
|
#81519
Could someone draw out the diagram as explained by Answer Choice B? Thank you.
 Robert Carroll
PowerScore Staff
  • PowerScore Staff
  • Posts: 1787
  • Joined: Dec 06, 2013
|
#81536
Hope,

I wish I could! But note what the question is asking: "Each of the following is possible EXCEPT". In other words, every wrong answer is something that it is possible to do. The correct answer, then, will be the one answer that is not possible. So no one can create a diagram that makes answer choice (B) work, and that's exactly why it's the correct answer for this question.

Let's talk a little more about why answer choice (B) can't happen.

As the explanation that starts this thread says, if answer choice (B) is true, then exactly three sessions would be attended by the employees, each session attended by a pair of employees. Let's think about those pairs. What if M and S never attend together? Then each attends once with T, and there are only two sessions possible (MT and ST). That's not allowed - we need exactly three sessions, as noted.

So let's try again. What if M and S attend together twice? Then T can never pair up with them, so, again, we will not have three sessions of two people each, as required.

So what must happen is that M and S attend together exactly once, then each attends something with T. So our pairs of people attending the three sessions are as follows: MS, MT, ST. T does nothing on the third day, so the MT and ST sessions are on the first and second days, in either order. Because M and S cannot attend investing, the MT and ST sessions must be on hiring and regulations; they can't be on the same topic, because then T would be repeating a topic.

So, at this point, we have MT attending hiring or regulations together on day 1 or day 2, and ST attending regulations or hiring together on day 2 or day 1. We need MS to attend something together. But it can't be on hiring - either the MT or the ST pair already did that, so one of the pair MS is not excluded from hiring. But it also can't be regulations for the same reason. So MS can't attend anything without violating some rule.

This is why answer choice (B) does not work.

Robert Carroll
User avatar
 KwakuS
  • Posts: 35
  • Joined: Jun 03, 2021
|
#96326
Thank you for the explanation. Is there a way to intuit this more quickly? I feel like working through all those hypotheticals would take way too long on the exam.

Thanks,
Kwaku
 Adam Tyson
PowerScore Staff
  • PowerScore Staff
  • Posts: 5153
  • Joined: Apr 14, 2011
|
#96359
In every game, Kwaku, you should give some thought to the numerical issues raised by the scenario and the rules, and this should happen while building your diagram. Here, there are a lot of numerical things to consider: 3 variables, each of which goes twice; 9 spaces in the base, meaning at least 3 will be empty; some variables might share a single space, reducing the total number of occupied spaces from a maximum of 6 to something lower and increasing the number of empty spaces.

With all that going on, it would be wise to consider minimums and maximums right from the start. It seems pretty clear that these 3 variables can occupy at most 6 spaces, and there are many ways that could happen and no need to draw out any of them. So turn your attention to the minimum. Could all three go together twice? Nope, that's against the rules. Could we do it in three pairs? Play with it and see, and soon you'll discover that's not possible, and that's how you find yourself fully prepared for this question. The answer is one you already prephrased at the beginning because you were thinking about the numbers, as you always should.

For this reason, you don't need to intuit anything about any of the other answers. You could look at all of them and say to yourself "I don't know, maybe." But when you see answer B, your reaction should be "oh yeah, that can't happen, because I already tried that and it didn't work. Winner!" It's not about intuition vs testing answers, but about being rewarded for having prepared yourself for the kinds of questions they usually ask.
User avatar
 ellenolmore
  • Posts: 5
  • Joined: Jun 03, 2023
|
#102225
I dont understand what A is saying, or why it is wrong. since it seems to not make sense then why is it not part of the "except" part?
 Luke Haqq
PowerScore Staff
  • PowerScore Staff
  • Posts: 747
  • Joined: Apr 26, 2012
|
#102277
Hi ellenolmore!

Another way of phrasing (A) is that no more than 1 employee attends any given session. According to this answer choice, it's possible that some sessions will have no employees, but if the session is attended, then a max of 1 employee attends the session.

This is possible. For example, T could take investing on day 1 and hiring on day 2; M could take hiring on day 1 and regulations on day 2; and S could take regulations on day 1 and hiring on day 3. On that distribution of employees over sessions, all of the sessions that are attended are attended by only one employee.

Get the most out of your LSAT Prep Plus subscription.

Analyze and track your performance with our Testing and Analytics Package.